还没有登录哦!

[Critical Reasoning]

试题详情

题目:

The higher the level of certain vitamins and minerals in the bloodstream, the better a person's lung function, as measured by the amount of air the person can expel in one second.  The lung function of smokers is significantly worse, on average, than that of nonsmokers.  Clearly, therefore, one way for smokers to improve their lung function is for them to increase their intake of foods that are rich in these helpful vitamins and minerals.
 
Which of the following is an assumption on which this argument depends?

选项:

A、Smokers are less likely than nonsmokers to have diets that are rich in vitamins and minerals.
B、The lung function of smokers whose diets are rich in those vitamins and minerals is generally better than that of nonsmokers with comparable diets.
C、People whose diets are deficient in those vitamins and minerals do not typically have other health problems in addition to diminished lung function.
D、Stopping smoking will not typically improve lung function more than any diet changes can.
E、Smoking does not introduce into the body chemicals that prevent the helpful vitamins and minerals from entering the bloodstream.

答案:

E

提问:

选了A。 问这个argument能成立的假设。就是说能推出这个结论还差条件,所以选了A。他们less likely 吃,所以应该多吃。

解答:

点赞0
阅读3585
解答: sysadmin

提问:

每日一题

解答:

点赞0
阅读4186
解答: 小智老师

提问:

选错了D选项,因为停止吸烟不能改变lung function 所以才通过diet来改变,E选项感觉不太对,麻烦我敬爱的英吉老师帮忙解答一下~谢谢老师!

解答:

点赞0
阅读3600
解答: 英吉老师

提问:

选了A。 问这个argument能成立的假设。就是说能推出这个结论还差条件,所以选了A。他们less likely 吃,所以应该多吃。

解答:

点赞0
阅读3586
解答: sysadmin老师

提问:

读下来,留了C和E,感觉C支持,E反对,选了C。请老师讲讲思路

解答:

点赞-1
阅读3616
解答: sysadmin老师

问个问题

点我领取
免费专项课程
在线咨询